7. Find the sum of 67 tenths and 35 hundredths. Give your answer as a decimal.

Answers

Answer 1

Step-by-step explanation:

since the tenth for the 67 is 6

and the hundredth for 35 is 5

then the sum

6/10+5/100

0.6+0.05

= 0.65

Answer 2

Answer:

7.05

Step-by-step explanation:

67 tenths is 6.7

35 hundredths is 0.35

6.7 + 0.35 = 7.05

Which means the answer is 7.05.


Related Questions

Write the percent as a decimal. 10. 57.4%

Answers

0.574
not sure why this need to be 20 characters but we’ll make it
Write the percent as a decimal?(57.4%)
Answer: 0.574

WHAT IS THE EXACT VALUE OF PI ?

Answers

Answer:

3.14159

Step-by-step explanation:

3.14159 is the correct answer

PLEASE HELP ASAP: On a snowy day 85% of the students of Wilson high school were present. If 572 of the students were present, how many students were enrolled at the school?

Answers

Answer:

rounds to 672

Step-by-step explanation:

This is a proportion question.

572     85

___ = ___

 x       100

Do 572 x 100 first to get 57,200.

Then divide 57,200 by 85 to get around 672 students.

However, I haven't done these kinds of questions in a long time, therefore I could be wrong.

On a snowy day 85% of the students of Wilson high school were present., There were 672 students enrolled at Wilson high school.

To find out how many students were enrolled at Wilson high school, we can use the information given in the question. We are told that 85% of the students were present on a snowy day, and that 572 students were present. To find the total number of students enrolled, we can set up a proportion: Present students / Total students = Percentage present / 100 Let's plug in the values we know: 572 / Total students = 85 / 100 To solve for the total number of students, we can cross-multiply and solve for Total students: 572 * 100 = Total students * 85 57200 = Total students * 85 Dividing both sides by 85: 57200 / 85 = Total students Total students = 672 Therefore, there were 672 students enrolled at Wilson high school.

To know more about  enrolled here

https://brainly.com/question/30086638

#SPJ2

PLEASE ANSWER THIS: Andrea had $20. She bought 4 paper pads at $1.19 each, 3 notebooks at $1.89 each, and also a book at $4.95. Now she wants to use the remaining money to buy as many 79-cent pens as possible. How many pens could she buy?

Answers

Answer:

5 pens

Step-by-step explanation:

Currently she has spent:

4(1.19) + 3(1.89) + 4.95 = 15.38

She has this amount left:

$20 - 15.38 = 4.62

So if she wanted to buy as many pens with the $4.62:

4.62/0.78 = 5.84

But you can't buy .84 of a pen, so she can buy 5 pens

Answer: 5 pens

Step-by-step explanation:

FACTS:

andrea has $20.

She bought:

4 paper pads for $1.19 each

3 notebooks for $1.89 each

1 book for $4.95

Will use the remaining money to buy pens that cost 79 cents ($0.79) each.

How many pens can she buy?

Step 1: Find how much she spent on the paper pads

1.19x4= 4.76

Ans= $4.76

Step 2: Find how much she spent on notebooks

3x1.89=$5.67

Ans= $5.67

Step 3: Find how much she has spent so far.

4.76 + 5.67 +4.95= $15.38

Step 4; Find how much she has left.

20-15.38= 4.62

Step 5: Find the number of pens she can buy.

She has $4.62 to buy pens that cost $0.79 (79 cents) each. how many pens can she buy?

To solve this, divide 4.62÷0.79. the decimal makes no difference in what operation you do.

4.62÷0.79= 5.84810126582 which rounds to 5.8 which rounds to 6.

Step 6: Check

REMEMBER:

pens cost $0.79 each

we're assuming she got 6 pens

in order for 6 to be correct, 6x0.79 MUST equal 4.62 or less because that's how money she had to spend on pens.

0.79x6= 4.74. It's wrong. In this case, it's always the number that comes before it, in this case 5.

Even though 5x0.79= 3.95 which is less than 4.62, it must be correct because it makes so much more sense to have some money left than spend more than what you even have. In questions like this, don't always rely on rounding.

Answer= 5 pens

move the dot -(-D) on the number line

Answers

Answer:

the answer is 1/2

Step-by-step explanation:

-(-D) = -(-1/2)

2 negatives make a positive so it would be 1/2 or in other cases, 0.5

Person above is incrorrect.

D= -1/2

So -(D)= 1/2

Then -(1/2)= -1/2

Find the sale price using the following numbers.
Original price: $180; Rate of discount: 15%

Please help me to answer this question!
TYSM!!!!!

Answers

Answer:

$180 - (15% * $180) = $153

Step-by-step explanation:

Answer:

$153

Step-by-step explanation:

Hello!

Original Price: $180

Original Percentage: 100% (multiplier = 1)

Discount: 15% (multiplier = 0.15)

Given that the Original Percentage is 100% and the Discount is 15%, we are trying to find 85% of the Original Price.

Sale Price: 85% of Original Price (multiplier = 0.85)

SP = 85% of OPSP = 0.85(180)SP = 153

The sale price is $153.

Let N be a number created by repeating any two identical three-digit strings. That is, N=ABCABC. For example, 123123, 023023, and 400400 are in the form.

Is it true or false that every such N is divisible by 13? If true, please explain. If not, provide a counter-example.

Answers

Answer:

true

Step-by-step explanation:

there is a nice rule for when a number is divisible by 13 :

starting with the lowest digit value (the last on the right) form blocks of 3 digits as numbers and give them alternating signs (starting with +).

if the last block (the last on the left) has less than 3 digits, just use the remaining ones as a number.

add all these numbers (with their associated signs).

if the number calculated by this (alternative) sum is divisible by 13 then the original number is divisible by 13.

if N = abcdefghi, then (ghi - def + abc) must be divisible by 13, so that N is also divisible by 13.

in our case, N = abcabc.

the block calculation is therefore abc - abc = 0, which is always divisible by 13.

so, any number formed like this must be divisible by 13.

FYI - a quick look at why that is :

N = abcabc =

= a×10⁵ + b×10⁴ + c×10³ + a×10² + b×10 + c =

= a×(10⁵ + 10²) + b×(10⁴ + 10) + c (10³ + 1) =

= a×100100 + b×10010 + c×1001

define d = 1001 = 77×13

and we get

a×d×100 + b×d×10 + c×d

a sum is divisible by a number, if every summed up term is divisible by that number.

since d is a factor in every term, and d is divisible by 13, so is every term and therefore the whole number.

so, every number in the form of abcabc is divisible by 13.

It is true that every number in the form N = ABCABC, where ABC represents a three-digit string, is divisible by 13.

Is it true or false that every such N is divisible by 13?

Let's consider the number N = ABCABC, where ABC represents a three-digit string. We can express N as follows:

N = 1000 * ABC + ABC

= 1001 * ABC

Notice that 1001 is divisible by 13, as 1001 = 13 * 77. Therefore, if ABC is any three-digit string, 1001 * ABC is divisible by 13.

Since N = 1001 * ABC, and 1001 is divisible by 13, it follows that N is also divisible by 13. This holds true for every number in the form ABCABC, where ABC is a three-digit string.

This pattern holds for any three-digit string repeated twice, ensuring that every number in the form ABCABC is divisible by 13.

Leaen more about divisible

brainly.com/question/29373718

#SPJ2


7. Tyrone is thinking of a number. In his number:

- The digit 4 has a value of 0.04
- The digit 3 is in both the ones and tens places.
- The digit 5 is in the tenths place.
- The digit in the ones place is 2 more than the digit in the thousandths place.
What is Tyrone's number?
____ ____ . ____ ____ ____

Answers

Answer:

1. 72.33.24. none5.none6.97. 08. 4

Step-by-step explanation:

72.33.44none .. 56.789

8.3.6 A dress originally priced at $80 is put on sale at 25% off. If 10% tax is added to the sale price, then what is the total cost of the dress?

Answers

Answer:

$66

Step-by-step explanation:

25% of 80 = 20

80 - 20 = 60

10% of 60 = 6

60 + 6 = 66

The answer will be $66

find the value of x and y in a triangle

Answers

Answer:

a) 49 degrees     b) 56 degrees

Step-by-step explanation:

a) Vertical angles are lines that intersect each other that always form the same angles on "opposite" sides. Angle x and the 49 degrees are vertical angles, which means that x also has to be 49 degrees.

- - -


b) We know that any triangles' three internal angles combined equal 180 degrees.

Since we already know two out of three angles in the triangle (63 and 49) we can find the last unknown angle: 180 - (63 + 49) = 68. The unknown angle in the triangle is 68 degrees.

We also know that all straight lines are 180 degrees. The line that angle y rests on is 180 degrees. So we can subtract the 68 degrees from 180 degrees to find that y = (180 - 68) / 2, because angle y is the same on the other side of the 68 degrees.

So:
y = (180 - 68) / 2
y = 112 / 2
y = 56

Angle y is 56 degrees.



Apologies if my explanation is confusing.

Answer:

a) x = 49° C

b) y = 48° C

Step-by-step explanation:

a) x is 49° C because we know that opposite angles are the same which means that x is opposite to 49° C which makes it also 49° C.

b) y is 48° C because angles in a Triangle always add up to 180° C which means that 180 - 83 - 49 = 48 which makes the answer 48° C.

Josh spent 8 hours in the gym last week.His goal was to spend 12 hours in the gym. What percent of the goal did he meet?

Answers

If Josh spent 8 hours out of 12 hours, that would be 8/12 as a fraction. Then you would simplify that to 2/3 which equals 66%

Answer:

2/3 or 66%

Step-by-step explanation:

If you divide 12 into 3 groups you get 4. 4 + 4 = 8 therefore 8/12 = 66%

PLEASE ANSWER ASAP: Miriam bought 64 pounds of popcorn she kept 1/4 for her family then gave 1/8 of the remainder to her club and distributed the rest equally among 8 friends. How many ounces did each friend get?

Answers

Answer:

84 ounces

Step-by-step explanation:

Total pounds of popcorn bought = 64 pounds

1/4th kept for family ==> 1/4 * 64 = 16 lbs

Remainder = 64-16 = 48 lbs

1/8th of remainder to club ==> 1/8 * 48 = 6 lbs to club

Remainder left over = 48-6 = 42 lbs

Distributed 42 lbs between each of 8 friends equally
Each friend gets 42/8 lbs = (42/8) * 16 = 84 ounces

Since there are 16 ounces in a pound

From the given information each friend receives 84 ounces of popcorn.

Let's solve the problem using thee concept of Fractions

1. Miriam bought 64 pounds of popcorn.

2. She kept 1/4 of the popcorn for her family. So, the amount of popcorn left after this step is [tex]\(64 \times \frac{3}{4}\)[/tex].

Now, let's calculate the popcorn remaining after Miriam keeps some for her family:

[tex]\(64 \times \frac{3}{4} = 48\)[/tex] pounds.

Next, Miriam gave 1/8 of the remaining popcorn to her club, leaving her with [tex]\(48 \times \frac{7}{8}\)[/tex] pounds.

Now, let's calculate the popcorn remaining after Miriam gives some to her club:

[tex]\(48 \times \frac{7}{8} = 42\)[/tex] pounds.

Finally, she distributed the rest equally among 8 friends. To find out how much each friend gets, we'll divide the total popcorn left by the number of friends:

Amount each friend gets =[tex]\(\frac{42}{8}\)[/tex] pounds.

The problem asks for the answer in ounces, not pounds.

To convert pounds to ounces, we know that 1 pound is equal to 16 ounces. So, to get the amount of popcorn each friend gets in ounces, we'll multiply the previous answer by 16:

Amount each friend gets = [tex]\(\frac{42}{8} \times 16\)[/tex] ounces.

Amount each friend gets = [tex]\(84\)[/tex] ounces.

So, each friend will get 84 ounces of popcorn.

In summary, Miriam's 64 pounds of popcorn were reduced to 42 pounds after sharing with her family and club. Finally, the remaining popcorn was divided equally among her 8 friends, with each friend receiving 84 ounces.

To know more about Fractions here

https://brainly.com/question/29186998

#SPJ2

I just need help with this question

Answers

Answer:

0

Step-by-step explanation:

-2 + 2 = 0

The sum of a number and its negative self is always 0 .

Hope this helped and brainliest?

Answer: 0

Step-by-step explanation:

It asks for the sum of the points plotted. The given points are on 2, and -2.

The sum is the total you get when adding 2 or more numbers.

-2+2= 0

Hope this helps!

8.46 If 100 is decreased by a% and the result is decreased by b%, then what is the total percent decrease (in terms of a and b)? (You can assume that both a and b are between 0 and 100.)

Answers

Answer:

The total percent decrease is [tex]$a+b-\frac{a b}{100}$[/tex]

Step-by-step explanation:

We are given with a word problemWe are asked to find the total percentage decreaseWe can do this in two steps

         Step 1: Finding the two percentage decrease

         Step 2: Finding their difference

Step 1 of 2

Let the decrease by a% be 100-a

Then b% decrease is

[tex]$$\begin{aligned}\frac{b}{100} \times(100-a) &=\frac{100 b-a b}{100} \\\frac{100 b}{100}-\frac{a b}{100} &=b-\frac{a b}{100}\end{aligned}$$[/tex]

Step 2 of 2

The percentage difference between the two percentages is,

[tex]$$\begin{aligned}&(100-a)-\left(b-\frac{a b}{100}\right)=100-a-b+\frac{a b}{100} \\&100-a-b+\frac{a b}{100}=100-\left(a+b-\frac{a b}{100}\right)\end{aligned}$$[/tex]

Since 100 is the total percent [tex]$a+b-\frac{a b}{100}$[/tex] is the percentage decrease.

answer: a+b-ab/100 hope this is right

two numbers are the ratio of 2:7. their sum is 63. what is the larger number?

Answers

Answer:

49

Step-by-step explanation:

step 1: Add the ratios 2:7 which is 2+7=9

step 2: divide the bigger ratio (7) by the total calculated in step 1 (9) and multiply by the total number given (63)....

step 3: we have 7/9×63 which is 49

therefore the bigger number is 49

The larger number is 49

PLEASE ANYONE: The percent of change when a new value is larger than the original value is called a percent of _________?

A. Change
B. Decrease
C. Increase
D. Interest

Answers

Answer:

C. Increase

Step-by-step explanation:

Since the new value is larger than the original value, there was an increase in the number, so it is called percent of increase.

Answer:

Increase

Step-by-step explanation:


Let's say that 50% of people liked ice cream in 2020. In 2022, 65% of people liked ice cream. There is an increase of people liking ice cream. So, it is called a percent increase.

Remember...

A) A percent change can be either an increase OR a decrease.

B) A decrease can only LOWER the percentage

So, it has to be C.

find the solution B=√2+√3-√2-√3

Answers

Answer:

[tex]B=\sqrt{2}[/tex]

Step-by-step explanation:

Original equation:

[tex]B = \sqrt{2+\sqrt{3}} - \sqrt{2 - \sqrt{3}}\\[/tex]

Square both sides:

[tex]B^2=(\sqrt{2+\sqrt{3}})^2 + 2(\sqrt{2+\sqrt{3}} * (-\sqrt{2 - \sqrt{3}}) + (-\sqrt{2-\sqrt{3}})^2[/tex]

Cancel out the square roots and squares:

[tex]B^2=2+\sqrt{3}}+ 2(\sqrt{2+\sqrt{3}} * (-\sqrt{2 - \sqrt{3}}) + 2-\sqrt{3}[/tex]

Add the sqrt(3) and -sqrt(3) as well as 2 and 2

[tex]B^2=4 + 2(\sqrt{2+\sqrt{3}} * (-\sqrt{2 - \sqrt{3}})[/tex]

Use the identity: [tex]\sqrt{a} * \sqrt{b} = \sqrt{a * b}[/tex] to rewrite the two square roots being multiplied:

[tex]B^2=4 + 2(-\sqrt{(2+\sqrt{3}) * (2 - \sqrt{3}}))[/tex]

Use difference of squares: [tex](a-b)(a+b) = a^2-b^2[/tex]

[tex]B^2=4 + 2(-\sqrt{2^2-\sqrt{3}^2})[/tex]

Square both sides

[tex]B^2=4 + 2(-\sqrt{4-3})[/tex]

Subtract:

[tex]B^2=4 + 2(-\sqrt{1})[/tex]

Evaluate square root of 1:

[tex]B^2=4 + 2(-1)[/tex]

Multiply

[tex]B^2=4 -2[/tex]

Subtract

[tex]B^2=2[/tex]

Take square root of both sides:

[tex]B=\sqrt{2}[/tex]

b= 0 because you take the square root of 2 + the square root of 3 and then subtract that by the square root of 2 and then subtract that by the square root of 3 it will give you zero

*Decimal word problems really easy*

1. You need to take 3 grams of pain medication. You've already taken 0.75 grams of pain medication from a multi-purpose formula. How much more medication do you need?

2. Alicia paid 5.75 for a bag of pinto beans. The beans cost $.55 per lb. How much did the bag of pinto beans weigh?

3. A cake needs 15.284 grams of flour. You already have 1/8 of it. How much more flour do you need?

4. Keith is making a spice rack for his mom in woodworking class. He starts with a board that is 31.25 inches long. Then he cuts it into 5 equal pieces for the shelves. How long will each shelf be?

5. Greg's scout troop is organizing a pancake breakfast to raise money for their next camping trip. They are expecting a large crowd, so Greg buys 4 gallons of maple syrup. If the maple syrup costs 35.75 per 2 gallons, how much does he spend?

Answers

Answer:

For the first problem, it would be 2.25 grams.

For the second problem, it would be 10.45 pounds.

For the third problem, it would be 13.3735 more grams of flour

For the fourth problem it would be 6.25 inches long.

Step-by-step explanation:

1st problem:

3 - 0.75 = 2.25

2nd problem:

5.75 / .55 = 10.45454545 (so if we simplified it, then it would be 10.45)

3rd problem:

15.285 * 7/8 = 13.3735

4th problem:

31.25 / 5 = 6.25

I see that someone has already answered 1-4 so i will answer 5

so for 2 gallons it is 35.75 so to find out the amount for 4 gallons you would add 35.75+35.75 to get your answer of 71.5

hope this helped :>

Evaluate.

4 to the power of 3 + (-2) to the power of 4

Answers

Answer:

4

Step-by-step explanation:

3+(-2) is the same as 3-2 which is 1. and since with multiple exponents you work from the top you have 4 to the power of 1^4. 1^4 is 1, so it's 4^1, which is 4.

the answer is four.

Being sure with my Answers
Ali caught 28 birds. He sold out 8 of themand kept the rest equally in 4 cages. Hiw many birds were kept in each cage?

Answers

Answer: 5

Explanation: 28-8=20. 20/4= 5.
5 would be kept in each cage because 28-8=20 and 20/4= 5

PLEASE HELP: On the assembly line, Max can process 15 engines in 40 mins. How many engines can he process in 2 hours?

Answers

Answer:

He can process 45 engines in 2 hours.

Step-by-step explanation:

2 hours = 120 minutes

You can multiply 3 to 40 because 40x 3 = 120, but you have to do the same to 15, so 15 x 3 = 45.

Or you can divide the engines (15) by minutes (40) to find out how many engines can be processed per minute.

15/40 = 0.375 an engine per minute.

Then multiply that by 120, which is two hours in minutes.

0.375 x 120 = 45

45 engines in 2 hours

On a scale drawing 2 centimeters represent 50 meters. How many centimeters represent 5 meters?

Answers

Answer:

0.2 centimeters

Step-by-step explanation:
Strategy 1:

Make the equation: 5=a•s, where s is the scale factor and "a" is the number of centimeters which 5 meters

First, we need to find the scale factor. 50 meters would be s times greater than 2 centimeters. Find s by first converting meters into centimeters
50m=5000cm. 5000cm=2cm*s, so s = 2500
5m=a*2500, a=0.002 meters, and 0.002 meters is equal to 0.2 centimeters.

Strategy 2:
the ratio of the scale drawing to real life will always stay the same, so
2 cm / 50 meters = x / 5 meters, and cross multiply to get
x cm* 50 m = 2 cm * 5 m, so
x cm * 10 = 2 cm
x cm = 2/10 cm
x cm = 0.2 cm

Strategy 3:
Notice that 5 meters is 10 times smaller than 50 meters, so on the scale drawing, we are looking for a number 10 times smaller than 2 centimeters, so  2/10=0.2 cm

Answer:

0.2

Step-by-step explanation:

2 = 50

x = 5

50/5 = 10

do the same on the other side

so 2/10 = 0.2

8.26 During the softball season, Judy had 35 hits. Among her hits were 1 home run, 1 triple, and 5 doubles. The rest of her hits were singles. What percent of her hits were singles?

Answers

Answer:

1 home + 1 triple + 5 double

1 +3+5×2

1+3+10=14

35-14 =21 singles

21/35×100=60%

80% of her hits were singles

A box measures 1/2 ft. long 1/2 ft. wide, and 1 ft. high. Find the volume in
inches. Do not include units in your answer.

Answers

Answer:

1/4

Step-by-step explanation:

V = LWH

V = (1/2 ft)(1/2 ft)(1 ft)

V = 1/4 ft³

Answer:

432 in³

Explanation:

Given measurements:

L = 1/2 ftW = 1/2 ftH = 1 ft

To convert feet to inches, multiply by 12

L = 1/2 × 12 = 6 inchW = 1/2 × 12 = 6 inchH =  1 × 12 = 12 inch

Volume of rectangular prism:

⇒ L × W × H

⇒ 6 × 6 × 12

⇒ 432 in³

Please Help!
Colour key:
Yellow: Questions I have answered
Green: Information needed to answer questions
Blue: Questions I need help with
Please, feel free to correct me, Im not good at Surface Area

Answers

Answer:

2. It will be a 7 3/4in x 12 1/4in rectangle

4.  31,987 in^2

6. 30 547/640 in^3

Step-by-step explanation:

2. This is a cross-section. In order to get the cross-section, you take your shape. (a 7 3/4 x 12 1/4 x 3 3/4 rectangle) and essentially you remove the last dimension (Height).

4. Take your surface area value (319 35/64) and multiply it by 100 (10 per dimension length and width). This leaves you with 319.87x100 = 31,987 in^2

6. (308 35/64)/10. Pretty easy, take 308.546875, divide by 10, and you get 30.8546875, which is 30 547/640 in^3 in fractions.

Answer:

2. It will be a 7 3/4in x 12 1/4in rectangle

4.  31,987 in^2

6. 30 547/640 in^3

Step-by-step explanation:

2. This is a cross-section. In order to get the cross-section, you take your shape. (a 7 3/4 x 12 1/4 x 3 3/4 rectangle) and essentially you remove the last dimension (Height).

4. Take your surface area value (319 35/64) and multiply it by 100 (10 per dimension length and width). This leaves you with 319.87x100 = 31,987 in^2

6. (308 35/64)/10. Pretty easy, take 308.546875, divide by 10, and you get 30.8546875, which is 30 547/640 in^3 in fractions.

2 ACTIVITY: Estimating a Percents
Work with a partner. Use a model to estimate the answer to each question. c. 18 is what percent of 40?

Answers

Answer: 45%

Step-by-step explanation:

Converting the word problem into words, we have:

18= x/100*40

Solving for it, we get that x=45.

18 is what percent of 40?
Answer: 45%

Write each ratio as a fraction in lowest terms:
Q1 - 24.8 to 13.4
Q2 - 1.5 to 4.5

Answers

Answer:

124/67 and 1/3

Step-by-step explanation:

The second ratio is 1.5/4.5, which readily reduces to 1/3.

Dividing both numerator and denominator of 24.8 / 13.4 by 2 yields the simpler 12.4 / 6.7  or  124/67.  This latter result cannot be reduced.

7.28. Jason can type at a rate of 40 words per minute. How long will it take him to type a 2,000 word essay?

Answers

Answer:

Given:

Jason can type at a rate of 40 words per minute.

Find:

The time taken for him to type a 2000 words essay

Step-by-step explanation:

Using the Conversion factor.

Jason's typing essentially converts "minutes" into "words" and vice versa, so we can use a conversion factor.

We need to arrange the data so that the "words" units cancel and we are left with "minutes" units:

time = 2000 words * 1 minutes/40 words

2000/40 minutes = 50 minutes

The time taken for him to type a 2000 words essay is 50 minutes.

I’m only writing this because brainly won’t let me put it THE ANSWER IS 50

Susan needs material for her school project. She buys 3.75 yards of material at $5.72 a yard. What is the total cost of the material? Round to the nearest cent.

Answers

Answer: $21.45

Step-by-step explanation:

FACTS:

1 yard costs $5.72.

She bought 3.75 yards

Total cost= cost per yard x number of yards

Total: t

t= 5.72x3.75

t= $21.45

She spent $21.45 for 3.75 yards of material

$21.50 is the total material

Help me pleaaseee!!!!!!! I'll give you brainliest if theres another answer on this.

Answers

Answer:

Please see attached image

Step-by-step explanation:

it would be.
7
6
3
2
1
4
5
Other Questions
Which of the following are duties of the Federal Reserve System?(Choose 5)Group of answer choicesActs as the governments bankMaintains currencyProvides investment advice to saversSells mortgages to borrowersConducts monetary policyProvides credit cards to consumersSupervises banksMaintains the payment system What is the current flowing through the circuit shown? (V= 120 V, R = 20 02,R= 50 Q, R3= 1022) (Ohm's law: V = IR)RwwwwRwwR 3 [ 6 - ( + 2 ) ]= solve How long is the blue ribbon if the red ribbon is 12 inches?Tyler has 3 times as many books as Mai.How many books does Mai have if Tyler has: Stuart wants to raise $100 for the Rainbow Club charity. He already has three donations of $30.20.$10.50 and $5.00. How much does Stuart still need to raise? Which of the following is true of an effective scientific model (The) __________ is the adaptable tool used to create and deliver appealing combinations of appealing values along with meaningful and persuasive marketing messages. Three strings, attached to the sides of a rectangular frame, are tied together by a knot as shown in the figure. The magnitude of the tension in the string labeled C is 56.3 N. Calculate the magnitude of the tension in the string marked A. (You'll need to get the various positions from the graph. The ends of the strings are exactly on one of the tic marks.)331.35 N is incorrect. Three ropes A, B and C are tied together in one single knot K. (See figure.)If the tension in rope A is 65.3 N, then what is the tension in rope B?46.17 N is incorrect. You look in the sky and see two jetliners that you know are of equal size, yet one appears to be much larger. Because of your knowledge of __________, you will assume that the smaller jetliner is farther away A hammer of mass m = 0.46 kg is moving horizontally at a velocity of v = 6.5 m/s when it strikes a nail and comes to rest after driving the nail a distance x = 1.1 cm into a board.What is the duration of the impact, assuming the acceleration is constant during this time period, in terms of the given variables? It is not accepting numerical values What was the average force exerted on the nail, in terms of the mass, initial velocity, and distance traveled? What was the average force, in newtons, exerted on the nail? Indicators are definite signs that abuse or neglect has taken place What is the horizontal asympote of this graph? Repetition, practice, collaborative learning, and explicit instruction in English vocabulary all contribute to ASAP WILL GIVE BRAINLEST You find an error on your credit report: your credit card account indicates that you are 60 days late on your payment but you have bank records indicating that you have always made on-time paymentsWhat should you do FIRSTA.)Contact the credit card company to have them fix it B.)Contact the credit reporting agency C.)Wait a few weeks to see if it gets resolved D.)Call the police to report the possible identity theft Karla tanner opened a web consulting business called link works and completed the following transaction in thr first month of operations A group of psychologists study human, mental, and physical growth from the prenatal period through childhood, adolescence, adulthood, and old age. They are most likely to be ________ psychologists. Identify the segments that are parallel, if any, if A 520 Hz tone is sounded at the same time as a 516 Hz tone. What is the beat frequency? Please help i'm being timed! (max points)Write an essay in complete sentences; was manifest destiny a positive or negative event? Choose a position and defend it.